LSAT and Law School Admissions Forum

Get expert LSAT preparation and law school admissions advice from PowerScore Test Preparation.

User avatar
 Dave Killoran
PowerScore Staff
  • PowerScore Staff
  • Posts: 5848
  • Joined: Mar 25, 2011
|
#43484
Complete Question Explanation
(The complete setup for this game can be found here: lsat/viewtopic.php?t=7431)

The correct answer choice is (B)

The condition in the question stem assigns G and K to T and S:
J95_Game_#4_#22_diagram 1.png
This information eliminates answer choice (A).

The application of the third rule eliminates answer choices (C), (D), and (E).

Thus, answer choice (B) is correct.

Get the most out of your LSAT Prep Plus subscription.

Analyze and track your performance with our Testing and Analytics Package.